Clinical Judgment Practice Exam

अब Quizwiz के साथ अपने होमवर्क और परीक्षाओं को एस करें!

An antacid is prescribed for a client with gastroesophageal reflux (GERD). The client asks the nurse, "How does this help my GERD?" Which is the best response by the nurse? A. "Antacids decrease the production of gastric secretions." B. "It will improve the emptying of food through your stomach." C. "Antacids will neutralize the acid in your stomach." D. "This medication will coat the lining of your esophagus."

"Antacids will neutralize the acid in your stomach." Rationale: Antacids neutralize hydrochloric acid in the stomach reducing the heartburn associated with gastroesophageal reflux disease (GERD).

A client with delusions tells the nurse, "You aren't doing your job. Go get those people over there and shoot them before they get me." Which statement is the nurse's best response? A. "There is no one who will hurt you." B. "You are in a safe place. No one can get to you here." C. "You seem quite frightened right now." D. "What would you like to see me do to protect you?"

"You seem quite frightened right now." Rationale: A client with delusions firmly holds false beliefs to be true, and it is best to acknowledge feelings related to the delusion. Reassuring statements such as, "You will be alright" are not effective for such clients.

The school nurse is preparing a teaching pamphlet in response to requests from parents regarding an outbreak of pinworms at the local preschool. Which information about the most commonly prescribed medication, mebendazole, should be included? A. A second dose of medication should be given in two weeks. B. Only children with perianal itching should take the medication. C. Insert the medication as a rectal suppository. D. It is safe for children of all ages to take this medication.

A second dose of medication should be given in two weeks. Rationale: Teaching about mebendazole, the drug of choice for pinworm infestation, should include a reminder that a second dose should be given in two weeks in order to prevent reinfestation.

The nurse has completed the diet teaching of a client who is being discharged following treatment of a leg wound. A high protein diet is encouraged to promote wound healing. Which lunch choice by the client indicates that the teaching was effective? A. A peanut butter sandwich with soda and cookies. B. A tuna fish sandwich with chips and ice cream. C. A salad with three kinds of lettuce and fruit. D. Vegetable soup, crackers, and milk.

A tuna fish sandwich with chips and ice cream. Rationale: In a high protein diet, a lunch with fish and dairy contains the highest amount of protein. For instance, four ounces of tuna contains 11 grams of protein, and ice cream 5 grams of protein per cup.

An older client with heart failure (HF), coronary artery disease (CAD), and hypertension (HTN), is receiving these daily prescriptions: atenolol, furosemide, and enalapril. Which assessments should the nurse include in evaluating the effectiveness of the medications? (Select all that apply.) A. Heart sounds. B. Blood pressure. C. Daily weight. D. Bowel sounds. E. Range of motion.

A,B,C Rationale: Changes in heart sounds may include the presence of adventitious sounds if HF is not controlled. Atenolol and enalapril are both used to reduce blood pressure. Furosemide increases urine output and daily weights provides objective data related to fluid balance.

A client with deep vein thrombosis (DVT) is receiving a continuous intravenous heparin infusion. The client now has tarry, black diarrhea and reports abdominal pain. Which actions should the nurse implement? (Select all that apply.) A. Monitor stools for presence of blood. B. Auscultate bowel sounds in all quadrants. C. Review last partial thromboplastin time results. D. Prepare to administer warfarin. E. Assess characteristics of pain.

A,B,C,E Rationale: A client receiving continuous heparin is at risk for bleeding, which is manifested by abdominal pain, diarrhea, and the presence of blood in the stool which is determined with guaiac testing of stools for occult blood, so the nurse should monitor for blood in the stool. The nurse should also monitor the bowel sounds in response to the client's report of abdominal pain. Partial thromboplastin time (PTT) results should be evaluated since the client is receiving heparin.

An older client is admitted in respiratory distress secondary to heart failure (HF), coronary artery disease (CAD), hypertension (HTN), and atrial fibrillation. Which nursing problems should the nurse include in this client's plan of care? (Select all that apply.) A. Fatigue. B. Fluid volume excess. C. Fluid volume deficit. D. Decreased cardiac output. E. Altered peripheral tissue perfusion.

A,B,D,E Rationale: Fatigue, fluid excess, decreased cardiac output, and altered tissue perfusion are concerns for this client. HF is characterized by a decreased cardiac output, which causes compensatory fluid retention resulting in an excess fluid volume. The ineffectiveness of the heart's pumping action in those with HF results in altered peripheral tissue perfusion and fatigue.

An older adult with a terminal illness is receiving hospice care and is having difficulty coping with feelings related to death and dying. Which interventions should the nurse include in this client's plan of care? (Select all that apply.) A. Teach client how to use guided imagery. B. Encourage family to visit frequently. C. Record the client's desire to live. D. Instruct client and family to reconsider end of life choices. E. Encourage family to bring the client old photographs.

A,B,E Rationale: Guided imagery, encouraging family presence, and reminiscing with old photographs help reduce anxiety and enhance coping for a client with a terminal disease.

The nurse is ready to insert an indwelling urinary catheter as seen in the picture. At this point in the procedure, which actions should the nurse take before inserting the catheter? (Select all that apply) A. Hold the catheter 3 to 4 inches (7.5 to 10 cm) from its tip. B. Complete perianal care with soap and water. C. Ask the client to bear down as if voiding to relax the sphincter. D. Secure the urinary drainage bag to the bed frame. E. Gently palpate the client's bladder for distention.

A,C Rationale: Prior to inserting a urinary catheter, the nurse should grasp the catheter 3 to 4 inches (7.5 to 10 cm) from the catheter tip with the dominant hand and ask the client to bear down.

An adult male reports that he recently experienced an episode of chest pressure and breathlessness when he was jogging. The client expresses concern because both of his deceased parents had heart disease and his father had diabetes. He lives with his male partner, is a vegetarian, and takes atenolol which maintains his blood pressure at 130/74 mmHg. Which risk factors should the nurse explore further with the client? (Choose all that apply.) A. History of hypertension. B. Homosexual lifestyle. C. Vegetarian diet. D. Excessive aerobic exercise. E. Family health history.

A,E Rationale: Based on the client's family history and medication for management of hypertension, the nurse should further explore these risk factors for ischemic heart disease.

A male client in the final stages of terminal cancer tells his nurse that he wishes he could just be allowed to die. The client states that he is tired of fighting this illness and is only continuing treatments because his family wants him to live. Which action should the nurse take? A. Request a consultation with the hospital social worker. B. Ask the chaplain to discuss death issues with the client. C. Arrange a meeting with the family, physician, and client. D. Notify the family that treatments have been discontinued.

Arrange a meeting with the family, physician, and client. Rationale: For the client with a terminal illness who is tired of fighting, the nurse should advocate for the client and arrange a meeting with the client, family and healthcare team to discuss the client's wishes.

A new member joins the nursing team and spreads books on the table, puts items on two chairs, and sits on a third chair. The members of the group are forced to move closer and remove their possessions from the table. What action should the nurse leader take? A. Move to welcome and accommodate the new person. B. Ask the new person to move belongings to accommodate others. C. Tell the new person to remove belongings because of limited space. D. Bring in additional chairs so that all staff members can be seated.

Ask the new person to move belongings to accommodate others. Rationale: By using assertive behavior (asking the person to move belongings), the leader observes the norms of personal space and values of all staff members. Moving and bringing in additional furniture are passive behaviors negating the needs of self and others. Demands can antagonize the aggressive new person and escalate behavior.

Which self care measure is most important for the nurse to include in the plan of care of a client recently diagnosed with type 2 diabetes mellitus? A. Diabetic diet meal planning. B. Blood glucose monitoring. C. A realistic exercise plan. D. Self-injection techniques.

Blood glucose monitoring. Rationale: Monitoring blood glucose levels using a fingerstick glucose monitoring device is the most important self-care measure for a client with type 1 or type 2 diabetes mellitus.

When conducting diet teaching for a client who is on a postoperative full liquid diet, which foods should the nurse encourage the client to eat? (Select all that apply.) A. Canned fruit cocktail. B. Creamy peanut butter. C. Vegetable juice. D. Vanilla frozen yogurt. E. Clear beef broth.

C,D,E Rationale: A full liquid diet includes all liquids that are not clear, such as vegetable juice and frozen yogurt, as well as clear liquids.

The nurse identifies ventricular fibrillation on the telemetry central monitor of a client who was admitted with angina. The unlicensed assistive personnel (UAP) obtains the crash cart and defibrillator, and the nurse enters the client's room and begins cardiopulmonary resuscitation. Which action should the nurse direct the UAP to take next? A. Open the crash cart for IV equipment. B. Begin administration of 100% oxygen. C. Continue cardiac compressions. D. Shock the client with the defibrillator.

Continue cardiac compressions. Rationale: The Adult Cardiac Arrest algorithm states that chest compression should be given "hard and fast" with minimal interruptions to compressions. The UAP should assume the continuation of cardiac compressions while the nurse prepares advanced life support interventions and delivers cardiac defibrillation.

A client with osteoporosis related to long-term corticosteroid therapy receives a prescription for calcium carbonate. Which of this client's serum laboratory values requires intervention by the nurse? A. Total calcium 9 mg/dL (2.25 mmol/L SI) B. Creatinine 4 mg/dL (354 micromol/L SI) C. Phosphate 4 mg/dL (1.293 mmol/L SI) D. Fasting glucose 95 mg/dL (5.3 mmol/L SI)

Creatinine 4 mg/dL (354 micromol/L SI) Rationale: The client's creatinine level is elevated (normal is 0.6 to 1.2 mg/L [53 to 106 micromol/L SI]), indicating kidney impaired kidney function, which requires further evaluation to decrease the possibility of permanent kidney damage related to calcium supplements.

A client is admitted reporting an acute onset of right flank pain and urinary urgency. Which assessment is most important for the nurse to obtain? A. Current body temperature. B. Numerical rated pain intensity. C. Amount of daily caffeine intake. D. Fluid intake for past 24 hours.

Current body temperature. Rationale: For a client who is exhibiting signs of pyelonephritis, a temperature elevation would help to confirm this diagnosis. The nurse should also assess the client's perception of pain using a pain scale, especially related to kidney disease, but pain intensity assessment does not have the priority of body temperature.

A middle-aged male client, admitted to a critical care unit several weeks ago because of serious injuries sustained in a motor vehicle accident, is currently in stable condition. Based on this client's age and recent life-threatening crisis, which intervention is should the nurse implement? A. Provide a routine schedule of activities to facilitate trust. B. Encourage the client to reflect on personal goals and priorities. C. Discuss the cause of the accident with the client and his family. D. Allow long periods of uninterrupted rest in order to reduce fatigue.

Encourage the client to reflect on personal goals and priorities. Rationale: Critical illness often prompts a re-examination of one's life priorities. The individual faces mortality, perhaps for the first time. Based on Erikson's developmental theory, middle-aged individuals are at the stage of Generativity vs. Stagnation. Generativity involves reflection on one's accomplishments and a personal evaluation of success in meeting life goals.

During a postpartum assessment of a client who is 5 hours post vaginal delivery, the nurse determines the fundus is 3 finger breadths above the umbilicus and positioned to the client's left side. Which action should the nurse implement first? A. Encourage the client to void. B. Catheterize for residual urinary volume. C. Provide additional oral replacement fluids. D. Massage the fundus until firm.

Encourage the client to void. Rationale: During the immediate postpartum period, bladder distention prevents uterine contraction which predisposes the client to excessive uterine bleeding, so the client should void, which allows the uterus to contract and reposition midline between the umbilicus and the symphysis pubis. If the client is unable to void or completely empty the bladder, then catheterizing may be indicated.

The wife of a newly-diagnosed client with Parkinson's disease asks the nurse if alternative or complimentary medical therapies might cure the disease. Which response should the nurse provide? A. Explain that there are no known conventional, alternative, or complimentary therapies that cure Parkinson's disease. B. Tell the wife that her husband's neurologist would know more about alternative treatments to cure Parkinsonism. C. Encourage the wife to ventilate her feelings about having a husband with Parkinson's disease. D. Compile a list of alternative medications that are effective in curing Parkinson's disease.

Explain that there are no known conventional, alternative, or complimentary therapies that cure Parkinson's disease. Rationale: The client's wife should be given truthful information that there is no known cure for Parkinson's disease available today.

The nurse is conducting a visual screening of a group of older adults. Which finding should the nurse report to the healthcare provider immediately? A. Cloudy opacity of the crystalline lens. B. Gradual onset of continuous eye pain and blurred vision. C. Recent change in the ability to read and drive after dark. D. Gray-white circle around the iris of both eyes.

Gradual onset of continuous eye pain and blurred vision. Rationale: The onset of eye pain and blurred vision may indicate closed-angle glaucoma, which requires immediate medical intervention.

A young adult female presents at the emergency center with acute lower abdominal pain. Which assessment finding is most important for the nurse to report to the healthcare provider? A. Pain scale rating of a "9" on a 0 to10 scale. B. Last menstrual period was 7 weeks ago. C. Reports white, curdy vaginal discharge. D. History of irritable bowel syndrome (IBS).

Last menstrual period was 7 weeks ago. Rationale: Acute lower abdominal pain in a young adult female can be indicative of an ectopic pregnancy, which can be life-threatening. Since the client's last menstrual period was 7 weeks ago, a pregnancy test should be obtained to rule out ectopic pregnancy, which can result in intra-abdominal hemorrhage caused by a ruptured fallopian tube.

A client with bladder cancer had surgical placement of a ureteroileostomy (ileal conduit) yesterday. Which postoperative assessment finding should the nurse report to the healthcare provider immediately? A. Red edematous stomal appearance. B. Mucous strings floating in the drainage. C. Stomal output of 40 mL in last hour. D. Liquid brown drainage from stoma.

Liquid brown drainage from stoma. Rationale: The ureteroileostomy or ileal conduit is surgical created by implanting the ureters into an isolated segment of ileum that drains urine through an abdominal stoma. Brown drainage may be feces, which should not be coming out of the stoma and could indicate that the bowel was cut during the surgery, thereby putting the client at risk for sepsis.

The nurse administers morphine sulfate IV to a client with pancreatitis, who is experiencing extreme periumbilical pain and abdominal distention. Which additional intervention should the nurse implement? A. Apply heat to the abdomen. B. Initiate contact isolation. C. Provide a clear liquid diet. D. Maintain IV at 125 ml/hour.

Maintain IV at 125 ml/hour. Rationale: The client with pancreatitis should be kept NPO to rest the gastrointestinal tract, so hydration must be maintained with IV fluids, and IV access provides the most efficient route for medication administration.

The nurse is preparing a teaching plan for an older female client diagnosed with osteoporosis. Which expected outcome has the highest priority for this client? A. Identifies 2 treatments for constipation due to immobility. B. Names 3 home safety hazards to be resolved immediately. C. States 4 risk factors for the development of osteoporosis. D. Lists 5 calcium-rich foods to be added to her daily diet.

Names 3 home safety hazards to be resolved immediately. Rationale: A major goal for an older client with osteoporosis is maintenance of safety to prevent falls. The outcome is stated with the client actions that are specific and time oriented, such as the client names 3 home safety hazards to be resolved immediately.

The nurse completes auscultation of the thoracic region on an adolescent client. Which finding is considered normal for this client? A. Rhonchi. B. Absent sounds. C. Adventious sounds. D. No adventious sounds.

No adventious sounds. Rationale: To identify normal findings for an adolescent client, the nurse should perform auscultation of the thoracic region to differentiate normal physiological findings from abnormal findings. No adventitious sounds heard on auscultation of the thoracic region indicate normal function that is without structural abnormality, obstruction, or alteration in fluid dynamics, such as liquid or gas flow.

At 40-weeks gestation, a client who is in active labor is lying in a supine position and tells the nurse that she has finally found a comfortable position. What action should the nurse take? A. Encourage the client to turn on her left side. B. Place pillows under the client's head and knees. C. Explain to the client that her position is not safe. D. Place a wedge under the client's right hip.

Place a wedge under the client's right hip. Rationale: Hypotension from pressure on the vena cava due to the weight of the fetus is a risk for the full-term client. Placing a wedge under the right hip will displace the fetus and relieve pressure on the vena cava.

While administering a continuous insulin infusion to a client with diabetic ketoacidosis, it is essential for the nurse to monitor which serum lab value? A. Hemoglobin. B. Protein. C. Calcium. D. Potassium.

Potassium. Rationale: As insulin lowers the blood glucose of a client with diabetic ketoacidosis, the serum potassium level also decreases as potassium returns to the cell. This can cause potentially fatal hypokalemia, so it is essential for the nurse to monitor the client's serum potassium.

The nurse is discussing mitigation at a disaster preparedness committee meeting. Which activity should the nurse suggest to enhance mitigation? A. Provide a community disaster preparedness meeting. B. Discuss some ways to ensure safety in the home during a disaster. C. Participate as an active member of the local American Red Cross. D. Design requirements for an Incident Command Center.

Provide a community disaster preparedness meeting. Rationale: Mitigation is implementing actions or measures that can reduce the severity of a disaster's effects such as awareness and education. Ensuring safety in the community is an example of mitigation.

An older client with a history of pernicious anemia has developed ataxia and paresthesia. In planning care, which nursing intervention has the highest priority? A. Provide assistance with ambulation. B. Keep the head of the bed elevated. C. Instruct about healthy diet choices. D. Offer a PRN sleep aid at night.

Provide assistance with ambulation. Rationale: Pernicious anemia, due to a vitamin B12 deficiency, may result in neurological deficits, such as confusion, paresthesias in the extremities, and ataxia, which places the client at risk for falls. The highest priority is to ensure that the client is assisted with ambulation.

A client with emphysema is being discharged from the hospital. The nurse enters the client's room to complete discharge teaching. The client reports feeling a little short of breath and is anxious about going home. What is the best course of action? A. Postpone discharge instructions at this time and offer to contact the client by phone in a few days. B. Invite the client to return to the unit for discharge teaching in a few days, when there is less anxiety. C. Provide only necessary information in short, simple explanations with written instructions to take home. D. Give detailed instructions speaking slowly and clearly while looking directly at the client when speaking.

Provide only necessary information in short, simple explanations with written instructions to take home. Rationale: Information is not retained when the recipient is anxious, and too much information can increase worry. Simple, short explanations should be provided.

A client recovering from pneumonia who has a history of severe chronic obstructive pulmonary disease (COPD) and peripheral vascular disease (PVD) is being discharged from a skilled nursing facility. Which action is most important for the nurse to implement? A. Explain exercise daily regimen. B. Reinforce need for adequate hydration. C. Demonstrate specific strengthening exercises. D. Provide typed instructions for healthy diet selections.

Reinforce need for adequate hydration. Rationale: For a client with chronic obstructive pulmonary disease (COPD) recovering from pneumonia, the priority intervention is maintaining adequate hydration, which helps mobilize secretions so that the client can productively cough, thereby decreasing the risk of reoccurring pneumonia.

A 6-month-old infant is admitted to the hospital with diarrhea. The mother is feeding the infant a bottle of tap water and tells the nurse that the baby has taken three 8-ounce bottles of water in the last 4 hours. Which laboratory finding is most important for the nurse to monitor? A. Creatinine clearance. B. White blood cell count. C. Serum potassium levels. D. Serum sodium levels.

Serum sodium levels. Rationale: Serum sodium levels should be monitored because the recent water intake places this infant at risk for water intoxication and hyponatremia.

When assessing a recently delivered, multigravida client, the nurse finds that her vaginal bleeding is more than expected. Which factor in this client's history is related to this finding? A. She received butorphanol 2 mg IVP during labor. B. The second stage of labor lasted 10 minutes. C. She is a gravida 6, para 5. D. She is over 35 years of age.

She is a gravida 6, para 5. Rationale: Repeated gravid experiences cause the uterus to lose muscle tone (uterine atony) which is the most common cause of excessive bleeding following childbirth.

A nurse stops at the site of a motorcycle accident and finds a young adult male lying face down in the road in a puddle of water. It is raining, no one is available to send for help, and the cell phone is in the car about 50 feet (15 meters) away. What action should the nurse take first? A. Stabilize the victim's neck and roll over to evaluate his status. B. Examine the victim's body surfaces for arterial bleeding. C. Open the airway and initiate resuscitative measures. D. Return to the car to call emergency response 911 for help.

Stabilize the victim's neck and roll over to evaluate his status. Rationale: To determine reponsiveness, the victim should be rolled while stabilizing the neck. The airway should be opened using a modified jaw thrust and airway, breathing, and circulation assessed. If alone the healthcare provider is rescuing an unresponsive victim that may be in asphyxial arrest, such as drowning, 5 cycles of CPR (2 minutes) should be provided before leaving the victim to phone the emergency response number.

A male client who is participating in an anger management assignment asks if he can make a leather belt in occupational therapy. The client begins pounding the leather vigorously with a mallet to imprint designs on the belt. Which defense mechanism is the client using? A. Sublimation. B. Regression. C. Compensation. D. Suppression.

Sublimation. Rationale: The client is channeling his socially unacceptable feelings of anger and impulsivity into socially acceptable activities (pounding leatherwork), which is called sublimation.

A client is scheduled for a spiral computed tomography (CT) scan with contrast to evaluate for pulmonary embolism. Which information in the client's history requires follow-up by the nurse? A. CT scan that was performed six months earlier. B. Metal hip prosthesis was placed twenty years ago. C. Takes metformin for type 2 diabetes mellitus. D. Report of client's sobriety for the last five years.

Takes metformin for type 2 diabetes mellitus. Rationale: Metformin may interact with iodinated contrast dye, causing acute renal failure and increased risk of lactic acidosis. Therefore, the nurse should consult with the healthcare provider regarding withholding the client's metformin at the time of the test and for at least 48 hours after the IV contrast media is given.

The nurse is caring for an adolescent who fell 20 feet (6.1 meters) 5 months ago while climbing the side of a cliff and has been in a sustained vegetative state since the accident. Which intervention should the nurse implement? A. Talk directly to the adolescent while providing care. B. Monitor vital signs and neuro status every 2 hours. C. Inquire about food allergies and food likes and dislikes. D. Initiate open communication with the teen's parents.

Talk directly to the adolescent while providing care. Rationale: Talking directly to the client who is in a sustained vegetative state provides environmental stimulation and includes him in an interpersonal relationship because he may still be able to hear and process verbal communication. Open communication that is compassionate and honest provides support to the family, but verbal stimulation is an important aspect of caring for comatose clients and offers hope for the possibility of a response.

The nurse is developing a plan of care for an older male client with type 2 diabetes who reports blurred vision. Which outcome should the nurse include in the plan of care for this client? A. The nurse will demonstrate the procedure for accurate eye care. B. The client will express acceptance of his changing health status. C. The client's family will state signs and symptoms about the disease. D. The client's daily blood pressure will be less than 140/80 mmHg this month.

The client's daily blood pressure will be less than 140/80 mmHg this month. Rationale: A client who is reporting blurred vision is likely experiencing complications related to type 2 diabetes, such as retinopathy or cardiovascular changes. The client's daily blood pressure will be less than 140/80 mmHg this month is an outcome statement in the plan of care that is client-centered, measurable, and has a time frame.

A mother brings her 4-month-old son to the clinic with a quarter taped over his umbilicus, and tells the nurse the quarter is supposed to fix her child's hernia. Which explanation should the nurse provide? A. This hernia is a normal variation that resolves without treatment. B. Restrictive clothing will be adequate to help the hernia go away. C. An abdominal binder can be worn daily to reduce the protrusion. D. The quarter should be secured with an elastic bandage wrap.

This hernia is a normal variation that resolves without treatment. Rationale: An umbilical hernia is a normal variation in infants that occurs due to an incomplete fusion of the abdominal musculature through the umbilical ring that usually resolves spontaneously as the child learns to walk.

A client who is admitted with an acute coronary syndrome (ACS) receives eptifibatide, a glycoprotein (GP) IIB IIIA inhibitor. Which assessment finding places the client at greatest risk? A. Blood pressure of 100/60 mmHg. B. Incontinent with blood in urine. C. Unresponsive to painful stimuli. D. Presence of hematemesis.

Unresponsive to painful stimuli. Rationale: Eptifibatide, a glycoprotein (GP) IIB IIIA inhibitor of platelet aggregation, is administered IV for acute coronary syndrome (ACS), and bleeding is its most significant side effect. A sudden onset of unresponsiveness may indicate intracranial bleeding, which is the most life-threatening finding related to bleeding.

The nurse is providing intermittent gavage feedings for a 33-week gestational age newborn. The nurse positions the newborn in a right side-lying position with the head slightly elevated and passes the feeding tube through the mouth. Prior to administering the bolus feeding, it is most important for the nurse to obtain which assessment? A. Nasal breathing obstruction. B. Gag reflex and vomiting. C. Volume of gastric residual. D. Intake and output.

Volume of gastric residual. Rationale: To avoid over-distending the stomach and create feeding intolerance, a common practice for premature infants is to deduct the amount of the residual gastric volume from the total prescribed bolus feeding. Before instilling the bolus feeding, the nurse should aspirate the stomach contents, measure it, return it, and deduct that amount from the next feeding.

The camp nurse is teaching adolescents about the prevention of tinea pedis. Which instruction should the nurse include in the teaching plan? A. Wear water shoes in the public shower. B. Do not share a brush or comb with anyone. C. Use moisturizing creams to retain skin moisture. D. Avoid using cosmetics that block sebaceous glands.

Wear water shoes in the public shower. Rationale: Wearing water shoes prevents the transmission of tinea pedis (Athlete's Foot) in a public shower.

When planning care for a client with acute pancreatitis, which nursing intervention has the highest priority? A. Withhold food and fluid intake. B. Initiate IV fluid replacement. C. Administer antiemetics as needed. D. Evaluate intake and output ratio.

Withhold food and fluid intake. Rationale: The pathophysiologic processes in acute pancreatitis result from oral fluid and food ingestion that causes secretion of pancreatic enzymes, which destroy ductal tissue and pancreatic cells, resulting in autodigestion and fibrosis of the pancreas. The main focus of nursing care is reducing pain caused by pancreatic destruction through interventions that decrease GI activity, such as keeping the client NPO.

What instruction should the nurse provide the parents of a 3-year-old boy with a BMI-for-age at the 97th percentile? A. Your child is tall for his age, so be sure he gets plenty to eat to stay healthy. B. Your child is overweight for his age and size, so help him select more healthy foods. C. Your child's weight is in the high range, but is probably normal for his body build. D. Your child has very strong bones, so continue to maintain the same diet.

Your child is overweight for his age and size, so help him select more healthy foods. Rationale: Children with a BMI-for-age at or above the 95th percentile are considered overweight, and at risk for obesity and all the associated health problems. The nurse should offer recommendations for healthy eating and exercise.

A young male client is admitted to rehabilitation following a right above-knee amputation (AKA) for a severe traumatic injury. He is in the commons room and anxiously calls out to the nurse, stating that his "right foot is aching." The nurse offers reassurance and support. Which additional intervention is most important for the nurse to implement? A. Offer to assist the client to a quieter location so he can relax. B. Teach the client how to wrap the stump with an elastic bandage. C. Encourage discussion of feelings about the loss of his limb. D. Administer a prescription for gabapentin, a neuroleptic agent.

Administer a prescription for gabapentin, a neuroleptic agent. Rationale: Neuropathic pain commonly occurs after nerves are severed during amputation, and the client should be reassured that he is "not crazy" because the missing limb hurts. These occurrences of pain of the amputated limb are often referred to as "phantom pains". The exact cause is not known, but it is believed to have something to with the severed peripheral nerves and their circuitry with the spinal cord and the brain. Adjuvant medications, such as a neuroleptic agent, are often helpful in reducing this type of pain.

A client presents at the emergency department reporting a raspy voice, cold intolerance, and fatigue. Laboratory tests indicate an elevated thyroid stimulating hormone (TSH) and low T3 and T4 levels. After the client is admitted to the telemetry unit, which intervention is most important for the nurse to implement? A. Assess for presence of non-pitting edema. B. Administer prescribed dose of levothyroxine. C. Offer additional blankets and a warm drink. D. Note client's most recent hemoglobin level.

Administer prescribed dose of levothyroxine. Rationale: In the negative feedback mechanism of hypothyroidism, a low level of thyroid hormone stimulates TSH production by the hypothalamus and results in an elevated TSH level, but the thyroid gland does not respond with adequate production of thyroid hormones (T3 and T4) to regulate basal metabolic rate. These serum hormone levels indicate the need to administer supplemental thyroid hormone, such as levothyroxine, as soon as possible to avert possible myxedema coma.

The healthcare provider changes a client's medication prescription from IV to PO administration and doubles the dose. The nurse notes in the drug guide that the prescribed medication, when given orally, has a high first-pass effect and reduced bioavailability. What action should the nurse implement? A. Continue to administer the medication via the IV route. B. Give half the prescribed oral dose until the provider is consulted. C. Administer the medication via the oral route as prescribed. D. Consult with the pharmacist regarding the error in prescription.

Administer the medication via the oral route as prescribed. Rationale: Bioavailability refers to the percentage of drug available in the systemic circulation. An increase in dosage is necessary to provide a therapeutic effect for oral medications with significantly reduced bioavailability.

The nurse reviews the laboratory findings of a client with an open fracture of the tibia. The white blood cell (WBC) count and erythrocyte sedimentation rate (ESR) are elevated. Before reporting this information to the healthcare provider, which assessment should the nurse obtain? A. Onset of any bleeding. B. Appearance of wound. C. Bilateral pedal pulse force. D. Degree of skin elasticity.

Appearance of wound. Rationale: An elevation of the white blood cell count (WBC) and erythrocyte sedimentation rate (ESR) reflect the presence of infection and inflammation. The wound should be assessed for erythema, edema, tenderness or increased pain, local heat or fever, and purulent exudate, so the wound's appearance can be described when the laboratory results are reported to the healthcare provider.

When administering brompheniramine maleate, an extended release antihistamine tablet, the nurse is told by the male client that he cannot swallow tablets. Which intervention should the nurse implement? A. Document the client's refusal to take the medication. B. Crush tablet and mix with small amount of pudding. C. Ask the pharmacist to send medication in liquid form. D. Document that the client cannot take the prescription.

Ask the pharmacist to send medication in liquid form. Rationale: For a client who states he cannot swallow tablets, most medications can be administered in liquid form. Therefore, the nurse should ask the pharmacist to send the medication in liquid form if available.

A recently hired nurse who is in orientation is assigned to the medical unit. The charge nurse observes the new nurse prepare to administer a unit of packed red blood cells as seen in the picture. Which action should the charge nurse take? A. Tell the nurse to take the client's vital signs and then start the transfusion. B. Assist the nurse in changing the intravenous tubing attached to the blood. C. Verify that a 22-gauge intravenous catheter is used for the transfusion. D. Assume responsibility for the care of the client during the blood transfusion.

Assist the nurse in changing the intravenous tubing attached to the blood. Rationale: Blood transfusions should be administered through tubing with an in-line filter and Y-connection that is connected to normal saline solution. This special tubing is not hanging, so the charge nurse should assist the new nurse in preparing the correct tubing. The transfusion should not be started until the correct equipment is in place.

The client provides three positive responses to items on the CAGE (Cut down, Annoyed, Guilty, Eye-opener) questionnaire. Which interpretation should the nurse provide the client? A. One positive response indicates the client should seek help with alcohol addiction. B. At least two positive responses are strongly suggestive of alcohol dependence. C. All responses to the CAGE questionnaire must be positive to suggest alcohol dependence. D. The CAGE questionnaire is a tool used to identify general substance abuse.

At least two positive responses are strongly suggestive of alcohol dependence. Rationale: The CAGE (Cut down, Annoyed, Guilty, Eye-opener) questionnaire assess alcohol addiction. The client should be confronted with results of the CAGE screening tool which indicates that any two positive responses to the four "yes/no" questions are strongly suggestive of alcohol dependence.

While removing staples from a client's postoperative wound site, the nurse observes that the client's eyes are closed and the client's face and hands are clenched. The client states, "I just hate having staples removed." After acknowledging the client's anxiety, which action should the nurse implement? A. Encourage the client to continue to verbalize the anxiety. B. Attempt to distract the client with general conversation. C. Explain the procedure in detail while removing the staples. D. Reassure the client that this is a simple nursing procedure.

Attempt to distract the client with general conversation. Rationale: Distraction is an effective strategy when a client experiences anxiety during an uncomfortable procedure such as staple removal from a postoperative site.

The nurse implements a primary prevention program for sexually transmitted diseases in a nurse managed health center. Which outcome indicates that the program was effective? A. Average client scores improved on specific risk factor knowledge tests. B. New screening protocols were developed, validated, and implemented. C. Clients who incurred disease complications promptly received rehabilitation. D. More than half of at-risk clients were diagnosed early in their disease process.

Average client scores improved on specific risk factor knowledge tests. Rationale: Primary prevention programs focus on promoting health and preventing diseases. Improved client score on risk factor knowledge tests is an effective outcome for a primary prevention program for sexually transmitted diseases.

The nurse is preparing a discharge teaching plan for a client who had a liver transplant. Which instruction is most important to include in this plan? A. Limit intake of fatty foods for one month after surgery. B. Notify the healthcare provider if edema occurs. C. Increase activity and exercise gradually, as tolerated. D. Avoid crowds for first two months after surgery.

Avoid crowds for first two months after surgery. Rationale: Cyclosporine immunosuppression therapy is vital in the success of liver transplantation and can increase the risk for infection, which is critical in the first two months after surgery. Fever is often the only sign of infection and is best minimized by avoiding viral, fungal, or bacterial exposure in a crowd.

A male client with multiple myeloma is admitted with pneumonia and pancytopenia. The nurse reviews the complete blood cell count findings and identifies a platelet count of 20,000 cells/mm3 (20 x 109/L). Which intervention should the nurse include in the client's plan of care? A. Pace activities between planned rest periods. B. Avoid intramuscular injections. C. Limit exposure to visitors with respiratory infections. D. Monitor intake and output.

Avoid intramuscular injections. Rationale: Multiple myeloma is a malignant proliferation of plasma B cells that infiltrate the bone marrow and inhibit hemopoiesis, so the nurse should implement thrombocytopenia precautions and avoid injections, which can result in intramuscular hematoma formation.

The nurse is developing an educational program for older clients who are being discharged with new antihypertensive medications. The nurse should ensure that the educational materials include which characteristics? (Select all that apply.) A. Written at a twelfth-grade reading level. B. Contains a list with definitions of unfamiliar terms. C. Uses common words with few syllables. D. Printed using a 12-point type font. E. Uses pictures to help illustrate complex ideas.

B,C,E Rationale: During the aging process, older clients often experience sensory or cognitive changes, such as decreased visual or hearing acuity, slower thought or reasoning processes, and shorter attention span. Educational materials for this age group should include a list of terms, such as medical terminology that a client may not know and use common words that express information clearly and simply. Simple, attractive pictures help hold the learner's attention.

An older adult client is admitted to the stroke unit after recovery from the acute phrase of an ischemic cerebral vascular accident (CVA). Which interventions should the nurse include in the plan of care during convalescence and rehabilitation? (Select all that apply.) A. Suction oral cavity every 4 hours. B. Place a bedside commode next to bed. C. Play classical music in room while client is awake. D. Measure neurological vital signs every 4 hours. E. Encourage family to participate in the client's care.

B,D,E Rationale: For the client recovering from a cerebral vascular accident (CVA), a bedside commode promotes safety. To evaluate the client's continued convalescence and rehabilitation, monitoring neurological status guides care, monitors client's progress, and identifies early signs of complications. Encouraging the family to participate in the client's care helps prepare for home care and demonstrates the value of family members providing participative care for the client.

An unconscious client is admitted to the intensive care unit and is placed on a ventilator. The ventilator alarms continuously and the client's oxygen saturation level is 62%. Which action should the nurse take first? A. Call respiratory therapy. B. Begin manual ventilation immediately. C. Monitor oxygen saturation levels every 5 minutes. D. Silence the alarm and call the technician.

Begin manual ventilation immediately. Rationale: Ventilators provide mechanical respirations. A constant alarm and low oxygen saturation indicates a malfunction or problem with the respirations being provided. The first action that must be taken is to begin manual ventilation until the problem has been resolved.

The nurse is planning care for a client who admits having suicidal thoughts. Which client behavior indicates the highest risk for the client acting on these suicidal thoughts? A. Expresses feelings of sadness and loneliness. B. Neglects personal hygiene and has no appetite. C. Lacks interest in the activities of family and friends. D. Begins to show signs of improvement in affect.

Begins to show signs of improvement in affect. Rationale: When a depressed client begins to show signs of improvement, it can be because the client has "figured out" how to be successful in committing suicide. Depressed clients, particularly those who have shown signs of potentially becoming suicidal, should be watched with care for an impending suicide attempt might be greater when the client appears suddenly happy, begins to give away possessions, or becomes more relaxed and talkative.

A mother brings her 2-month-old infant to the clinic for a well-baby appointment. The nurse obtains a history and conducts a physical assessment. Which finding requires the most immediate intervention? A. History of poor feeding and vomiting. B. A positive Ortolani maneuver. C. Mother describes infant as irritable. D. Bilateral retinal hemorrhages.

Bilateral retinal hemorrhages. Rationale: Bilateral retinal hemorrhages with no reported trauma or injury to an infant are usually evidence of shaken baby syndrome. Shaken baby syndrome occurs with forcefully shaking an infant or a toddler causing shearing of the brain which causes permanent brain damage and can lead to death.

A female client who is admitted to the mental health unit for opiate dependency is receiving clonidine 0.1 mg PO for withdrawal symptoms. The client begins to complain of feeling nervous and tells the nurse that her bones are itching. Which finding should the nurse identify as a contraindication for administering the medication? A. Muscle weakness. B. Apical heart rate 72 beats/minute. C. Hypertension. D. Blood pressure 90/76 mm Hg.

Blood pressure 90/76 mm Hg. Rationale: Clonidine, an alpha adrenergic blocking agent used to treat hypertension, is also administered for symptoms related to opiate withdrawal. If a client's systolic blood pressure is less than 90 mm Hg, the medication should not be given.

A 6-year-old child who had surgery yesterday absolutely refuses to use the incentive spirometer. Which intervention should the nurse implement? A. Ask the mother to assist when it is time to use the spirometer. B. Allow child to choose when to perform incentive spirometry. C. Contract with the child to use spirometer only after meals. D. Blow out lights, blow bubbles, and encourage child's laughing.

Blow out lights, blow bubbles, and encourage child's laughing. Rationale: Young children may cooperate more if the procedure becomes a game. Extended exhalation, such as blowing and laughing, triggers coughing and mobilizes fluid in the tracheobronchial tree, which is the objective of using an incentive spirometer.

The nurse working in the psychiatric clinic has phone messages from several clients. Which call should the nurse return first? A. A young man with schizophrenia who wants to stop taking his medications. B. The mother of a child who was involved in a physical fight at school today. C. A client diagnosed with depression who is experiencing sexual dysfunction. D. A family member of a client with dementia who has been missing for five hours.

A family member of a client with dementia who has been missing for five hours. Rationale: Safety is always the priority concern, and the family member of the missing client with dementia needs assistance with contacting authorities as well as psychological support during this time.

A client presents to the clinic with concerns regarding her left breast. Which assessment finding is most important for the nurse to report to the healthcare provider? A. Bloody discharge from the nipple. B. A fixed nodular mass with dimpling of skin. C. A slight asymmetry of the breasts. D. Multiple firm, round, freely moveable masses.

A fixed nodular mass with dimpling of skin. Rationale: A fixed nodular mass with dimpling of the overlying skin is a common finding during late stages of breast cancer and should be reported. Although other findings may be included in the report, a bloody discharge may indicate a benign condition, such as intraductal papilloma.

A nurse who usually works on a step-down unit is moved to work a 12-hour shift in the critical care unit. Which client is best for the charge nurse to assign to this nurse? A. A ventilated client admitted today with respiratory failure and respiratory acidosis. B. A ventilator dependent client with chronic obstructive pulmonary disease (COPD). C. A client who has a new onset diabetic ketoacidosis (DKA) and is on an insulin drip. D. A client admitted for a narcotic overdose who is ventilated and has respiratory alkalosis.

A ventilator dependent client with chronic obstructive pulmonary disease (COPD). Rationale: A ventilator dependent client with chronic obstructive pulmonary disease (COPD) is the best choice to assign to a nurse who is moved to work in a more complex level of care than usually assigned. Clients with more acute or complex needs should be assigned to critical care staff.

A middle-aged client, diagnosed with Graves' disease, asks the nurse about this condition. Which etiological pathology should the nurse include in the teaching plan about hyperthyroidism? (Select all that apply.) A. Graves' disease, an autoimmune condition, affects thyroid stimulating hormone receptors. B. Large protruding eyeballs are a sign of hyperthyroid function. C. Early treatment includes levothyroxine. D. T3 and T4 hormone levels are increased. E. Weight gain is a common complaint in hyperthyroidism.

A,B,D Rationale: Graves' disease is an autoimmune disorder in which thyroid stimulating antibodies activate thyroid simulating hormone (TSH) receptors causing an increased production of thyroid hormone (T3 and T4). Manifestations include exophthalmos and weight loss. Treatment includes medication to block synthesis of thyroid hormone, so levothyroxine is contraindicated.

The nurse is arranging home care for an older client who has a new colostomy following a large bowel resection three days ago. The client plans to live with a family member. Which actions should the nurse implement? (Select all that apply.) A. Assess the client for self care ability. B. Provide pain medication instructions. C. Call home care agency to set up oxygen. D. Request a home safety inspection. E. Teach care of ostomy to care provider.

A,B,E Rationale: For the client with a new colostomy returning to home care, the nurse must assess the client's ability to care for self in order to make home care arrangements prior to discharge. Pain medication instructions, both verbal and written, should be provided prior to discharge. Teaching ostomy care to the care provider is essential, especially when the client is older and possibly confused.

A client is admitted for an exacerbation of heart failure (HF) and is being treated with diuretics for fluid volume excess. In planning nursing care, which interventions should the nurse include? (Select all that apply.) A. Observe for evidence of hypokalemia. B. Encourage oral fluid intake of 3,000 mL/day. C. Weigh the client daily, in the morning. D. Teach the client how to restrict dietary sodium. E. Monitor PTT, PT, and INR lab values.

A,C,D Rationale: For a client with heart failure and fluid volume overload, the goal is to decrease workload on the heart. Fluid volume excess is managed with diuretics to help eliminate the excess fluid load. Since many diuretics cause potassium excretion, the plan of care should include observing for hypokalemia and teaching the client how to restrict dietary sodium, which reduces fluid retention related to sodium intake. Fluid balance is best assessed by changes in daily weight.

A male client is admitted for the removal of an internal fixation device that was inserted for a fractured ankle. During the client's admission history, he tells the nurse that he recently received vancomycin for a methicillin-resistant Staphylococcus aureus (MRSA) wound infection. Which actions should the nurse take? (Select all that apply.) A. Collect multiple site screening cultures for MRSA. B. Call healthcare provider for a prescription for linezolid. C. Place the client on contact transmission precautions. D. Obtain a sputum specimen for culture and sensitivity. E. Continue to monitor the client for signs of an infection.

A,C,E Rationale: Until multi-site screening cultures come back negative, a client with a history of methicillin-resistant Staphylococcus aureus (MRSA), should be maintained on contact isolation to minimize the risk for hospital aquired infections. Monitoring the client for signs of an infection is a component of continuing perioperative care associated with the client's risk for residual MRSA colonization and possible nosocomial transmission.

A client with Type 2 diabetes mellitus is admitted for frequent hyperglycemic episodes and a glycosylated hemoglobin (A1C) of 10%. Insulin glargine 10 units subcutaneously once a day at bedtime and a sliding scale with insulin aspart every 6h are prescribed. What actions should the nurse include in this client's plan of care? (Select all that apply.) A. Fingerstick glucose assessments every 6h with meals. B. Mix bedtime dose of insulin glargine with insulin aspart sliding scale dose. C. Review with the client proper foot care and prevention of injury. D. Do not contaminate the insulin aspart so that it is available for IV use. E. Coordinate carbohydrate controlled meals at consistent times and intervals. F. Teach subcutaneous injection technique, site rotation, and insulin management.

A,C,E,F Rationale: The nurse should include fingerstick glucose assessments for insulin aspart sliding scale with meals, a teaching plan regarding infection risks related to chronic hyperglycemia, diet modification, and skills related to insulin management. Insulin glargine has a flat peak and should not be mixed with any other insulins.

A client with Myasthenia Gravis is admitted with bradycardia caused by an overdose of pyridostigmine. Which action should the nurse take first? A. Assess for excessive pulmonary secretions. B. Observe client for fasiculation and twitching. C. Administer a PRN dose of atropine 1 mg IV. D. Discontinue the next doses of pyridostigmine.

Administer a PRN dose of atropine 1 mg IV. Rationale: An overdose of pyridostigmine, a cholinesterase inhibitor drug, can precipitate a cholinergic crisis causing cholinergic-induced bradycardia. Atropine should be given to increase the heart rate and blood pressure.

A client with persistent low back pain has received a prescription for an electronic stimulator (TENS) unit. After the nurse applies the electrodes and turns on the power, the client reports feeling a tingling sensation. How should the nurse respond? A. Determine if the sensation feels uncomfortable. B. Decrease the strength of the electrical signals. C. Remove electrodes and observe for skin redness. D. Check the amount of gel coating on the electrodes.

Determine if the sensation feels uncomfortable. Rationale: Electronic stimulators, such as a transelectrical nerve stimulator (TENS) unit, have been found to be effective in reducing low back pain by "closing the gate" to pain stimuli. A tingling sensation should be felt when the power is turned on, and the nurse should assess whether the sensation is too strong, causing discomfort or muscle twitching.

The nurse assesses a client who had bilateral total knee replacements (TKR) four hours ago. The nurse notes that the dressing on the client's right knee is saturated with serosanguineous drainage. What action should the nurse implement? A. Withhold next scheduled dose of low molecular weight heparin. B. Confirm that the continuous passive motion device is intact. C. Determine if the wound drainage device is functioning correctly. D. Monitor the client's current white blood cell count (WBC).

Determine if the wound drainage device is functioning correctly. Rationale: An obstructed or non-functioning drainage device, which is placed in the operative site of the total knee replacement, can result in excessive drainage on the dressing. The nurse should first check the drainage device before taking further action.

To prevent infection by auto-contamination during the acute phase of recovery from multiple burns, which intervention is most important for the nurse to implement? A. Dress each wound separately. B. Avoid sharing equipment between multiple clients. C. Use gown, mask, and gloves with dressing change. D. Implement protective isolation.

Dress each wound separately. Rationale: Each wound should be dressed separately using a new pair of sterile gloves to avoid autocontamination (the transfer of microorganisms from one infected wound to a non-infected wound).

A client with chronic alcoholism is admitted with a decreased serum magnesium level. Which snack option should the nurse recommend to the client? A. Cheddar cheese and crackers. B. Carrot and celery sticks. C. Beef bologna sausage slices. D. Dry roasted almonds.

Dry roasted almonds. Rationale: Alcoholism promotes inadequate food intake and gastrointestinal loss of magnesium. Hypomagnesemia (less than 1.5 mEq/L or 0.75 mmol/L SI) can lead to cardiac dysrhythmias. Foods rich in magnesium include green leafy vegetables and nuts and seeds.

An older adult client is diagnosed with severe shingles and starts a new prescription for acyclovir, an antiviral medication. Which action should the nurse include during client teaching prior to discharge? A. Demonstrate how to apply sterile gauze dressings over the infected site. B. Explain the increased risk for postherpetic neuralgia during treatment. C. Encourage increased oral fluid intake while taking the medication. D. Schedule an appointment for medication peak and trough levels.

Encourage increased oral fluid intake while taking the medication. Rationale: Acyclovir is often used to treat shingles. The medication acyclovir can precipitate in the renal tubules, so increased fluid intake is beneficial while taking the medication.

A client is being treated for syndrome of inappropriate antidiuretic hormone (SIADH). On examination, the client has a weight gain of 4.4 lbs (2 kg) in 24 hours and an elevated blood pressure. Which intervention should the nurse implement first? A. Monitor daily sodium intake. B. Measure ankle circumference. C. Ensure client takes a diuretic every morning. D. Obtain serum creatinine levels daily.

Ensure client takes a diuretic every morning. Rationale: In syndrome of inappropriate antidiuretic hormone (SIADH), ADH is released resulting in fluid retention, dilutional hyponatremia, excess intravascular fluid, elevated blood pressure, and weight gain. Excessive fluid retention and intravascular blood volume can be managed with the administration of diuretics, which the nurse should ensure that the client takes each morning.

A client with generalized anxiety disorder does not want to communicate with friends, smokes 2 to 3 packages of cigarettes a day, and describes difficulty concentrating at work. Which coping strategy should the nurse include in the plan of care? A. Focus on small achievable tasks, not taxing problems. B. Concentrate on and ventilate emotions when distressed. C. Relax and reduce the amount of effort to solve the problem. D. Analyze past hurts and resentments to identify the source.

Focus on small achievable tasks, not taxing problems. Rationale: The client is manifesting maladaptive behaviors that prevent healthy adjustment and personal growth. Coping with life events can often feel overwhelming. The plan of care should include effective coping strategies that assist the client to focus on small achievable tasks, not taxing problems.

An adult woman with type 2 diabetes mellitus (DM2) is to be admitted within the next hour to the medical unit from the Emergency Department. The client's laboratory findings indicate that her serum glucose is 175 mg/dL (or 9.63 mmol/L - SI) and her A1c is 9%. When requesting a dinner tray for the client, which menu should the nurse select? A. Fried chicken breast, mashed potatoes, green beans, sliced tomatoes, and fresh apple pie. B. Grilled fish with whole-grain brown rice, steamed broccoli, and pear poached in red wine. C. Lean hamburger with cheese, tomato, and lettuce on whole-wheat bun, and angel food cake. D. Vegetarian lasagna with cheese and spinach, tossed green salad with ranch dressing, and fresh fruit.

Grilled fish with whole-grain brown rice, steamed broccoli, and pear poached in red wine. Rationale: A client's laboratory findings of a serum glucose of 175 mg/dL (or 9.63 mmol/L - SI) and an A1c of 9% indicate hyperglycemia and poor control of serum glucose over time. The best diet choice is one which has a low fat protein (grilled fish), whole-grains (brown rice), fresh vegetable (steamed broccoli), and fruit for dessert.

The nurse is developing the plan of care for a hospitalized child with von Willebrand's disease. What priority nursing intervention should be included in this child's plan of care? A. Reduce exposure to infection. B. Eliminate contact with cold drafts. C. Guard against bleeding injuries. D. Reduce contact with other children.

Guard against bleeding injuries. Rationale: Von Willebrand's disease is a hereditary bleeding disorder characterized by a deficiency of factor VIII (a defective serum protein called von Willebrand's factor) that results in a prolonged bleeding time because platelets fail to adhere to the walls of a ruptured vessel to form a platelet plug. The priority intervention to include in the plan of care is guarding against any injury that might result in bleeding from injection sites.

An adolescent who was diagnosed with diabetes mellitus Type 1 at the age of 9, is admitted to the hospital in diabetic ketoacidosis. Which occurrence is the most likely cause of the ketoacidosis? A. Incorrectly administered too much insulin. B. Skipped eating lunch. C. Ate an extra peanut butter sandwich before gym class. D. Had a cold and ear infection for the past two days.

Had a cold and ear infection for the past two days. Rationale: Acute infections increase the body's need for insulin to control hyperglycemia and put the client at risk for diabetic ketoacidosis (DKA).

When is it most important for the nurse to assess a pregnant client's deep tendon reflexes (DTRs)? A. Within the first trimester of pregnancy. B. If the client has an elevated blood pressure. C. When the client has ankle edema. D. During admission to labor and delivery.

If the client has an elevated blood pressure. Rationale: A systolic greater than 140 mm Hg or a diastolic greater than 90 mm Hg is a sign of preeclampsia. Deep tendon reflexes (DTRs) greater than 2+ are also a sign of preeclampsia as is proteinuria greater than or equal to 1+ on dipstick measurement. Preeclampsia usually occurs after the 20th week of pregnancy.

A client with a history of adrenal insufficiency is admitted to the intensive care unit with an acute adrenal crisis. The client is complaining of nausea and joint pain. Vital signs are: temperature 102o F (38.9o C), heart rate 138 beats/minute, blood pressure 80/60 mmHg. Which intervention should the nurse implement first? A. Administer PRN oral antipyretic. B. Infuse an intravenous fluid bolus. C. Cover client with cooling blanket. D. Obtain an analgesic prescription.

Infuse an intravenous fluid bolus. Rationale: Acute adrenal crisis is a life-threatening condition that is complicated by shock if untreated. When the blood pressure is low, IV fluids are essential.

The nurse is caring for a client who is receiving continuous ambulatory peritoneal dialysis (CAPD) and notes that the output flow is 100 mL less than the input flow. Which actions should the nurse implement first? A. Change the client's position. B. Irrigate the dialysis catheter. C. Check the client's blood pressure and serum bicarbonate. D. Continue to monitor intake and output with next exchange.

Change the client's position. Rationale: To facilitate drainage, for a client who is receiving continuous ambulatory peritoneal dialysis (CAPD), the client's position should be changed to move the dialysate solution toward the catheter if the output flow is less than the input flow.

A client with bleeding esophageal varices receives vasopressin IV. Which should the nurse monitor for during the IV infusion of this medication? A. Chest pain and dysrhythmia. B. Vasodilation of the extremities. C. Hypotension and tachycardia. D. Decreasing GI cramping and nausea.

Chest pain and dysrhythmia. Rationale: In large doses, vasopressin may produce increased blood pressure, coronary insufficiency, myocardial ischemia or infarction, and dysrhythmia.

A client with a lower respiratory tract infection receives a prescription for ciprofloxacin 500 mg PO every 12 hours. When the client requests an afternoon snack, which dietary choice should the nurse provide? A. Vanilla-flavored yogurt. B. Low-fat chocolate milk. C. Calcium-fortified juice. D. Cinnamon applesauce.

Cinnamon applesauce. Rationale: Dairy products and calcium-fortified dairy products decrease the absorption of ciprofloxacin. Cinnamon applesauce contains no calcium, so this is the best snack selection.

A client's subjective data includes dysuria, urgency, and urinary frequency. What action should the nurse implement next? A. Inquire about recent sexual activity. B. Instruct to wipe from front to back. C. Palpate the suprapubic region. D. Collect a clean-catch specimen.

Collect a clean-catch specimen. Rationale: The first step in addressing the subjective symptoms is to further assess. This client is exhibiting symptoms of a urinary tract infection (UTI), so the nurse should collect a clean-catch urine specimen for urine analysis, and culture and sensitivity. A urine analysis is the best method for determining the existence of a urinary tract infection (UTI).

The nurse is assigned to provide care for a client who is scheduled for a laparoscopic cholecystectomy in two hours, at 0900. What nursing action is most important? A. Review postoperative instructions with the client. B. Offer to assist the client to the restroom to void. C. Determine when the client last had pain medication. D. Confirm that the client has been NPO since midnight.

Confirm that the client has been NPO since midnight. Rationale: The client should be NPO prior to a surgical procedure, such as a laparoscopic cholecystectomy, to reduce risk for vomiting and aspiration.

A client with postpartum depression, who is admitted to the behavioral health unit, refuses to leave her room or eat meals. In addition to maintaining physical safety, which short-term goal should the nurse include in the plan of care? A. Sleeps at least 6 hours per night. B. Consumes 3 meals and 1500 mL of fluid per day. C. Engages in one client-to-client interaction daily. D. Attends one group activity per day.

Consumes 3 meals and 1500 mL of fluid per day. Rationale: Based on Maslow's Hierarchy of Needs Theory, nutrition and fluid intake are priority needs for a client during postpartum involution.

An adult male with end-stage liver disease has been unresponsive for the past 3 days. His electroencephalography (EEG) reveals no active brain activity. The family wants to discontinue feeding and donate his viable organs. Which action should the nurse to take? A. Contact the regional organ procurement agency. B. Convene a multidisciplinary care conference. C. Explain that the client may not be an organ donor candidate. D. Discontinue feedings and fluids per the family's request.

Contact the regional organ procurement agency. Rationale: The client's family should be referred to an organ procurement agency so that a trained worker can assist the them with the procurement process. The care conference should be mediated by the procurement agency. The agency is responsible for discussing with the family what organs are eligible for procurement, and they will also specify when nutrition and hydration should be discontinued.

A client with gestational diabetes is being induced for labor. Which assessment is most important for the nurse to perform prior to increasing the oxytocin rate? A. Vaginal exam. B. Blood pressure. C. Contraction pattern. D. Fingerstick glucose.

Contraction pattern Rationale: Before increasing the rate of the oxytocin infusion, it is most important for the nurse to determine the frequency and pattern of uterine contractions.

The healthcare provider prescribed furosemide for a 4-year-old child who has a ventricular septal defect. Which outcome indicates to the nurse that this pharmacological intervention was effective? A. Daily weight decrease of 2 pounds (0.9 kg). B. Blood urea nitrogen (BUN) increase from 8 to 12 mg/dL (2.9 to 4.3 mmol/L). C. Urine specific gravity change from 1.021 to 1.031. D. Urinary output decrease of 5 mL/hour.

Daily weight decrease of 2 pounds (0.9 kg). Rationale: Furosemide is a loop diuretic. Assessment of weight is usually an effective, non-invasive measure for evaluating fluid volume status. This client's weight loss indicates that the furosemide is removing excess fluid.

The nurse is assessing a client with a closed head injury sustained in a motor vehicle collision. Which finding indicates the lowest level of neurologic functioning? A. Withdrawal from painful stimuli. B. Decerebrate posturing during position changes. C. Localization of a tactile stimulus. D. Decorticate posturing during tracheal suctioning.

Decerebrate posturing during position changes. Rationale: The lowest level of neurological functioning is characterized by decerebrate posturing (abnormal extension). Posturing (decorticate or decerebrate) is not considered a purposeful response to pain. As neurological functioning deteriorates, the client will progress from localization of a tactile stimulus to withdrawal from painful stimuli, followed by decorticate posturing in response to the stimuli, before finally exhibiting decerebrate posturing.

After receiving IV fluids in the emergency department, an elderly client is admitted to the acute care unit with a medical diagnosis of dehydration. The client is receiving 0.9% normal saline at 125 mL/hour via a saline lock and has a bounding pulse, tachycardia, and pedal edema. When contacting the healthcare provider, the nurse anticipates a prescription for what intervention? A. Decrease the rate of the normal saline infusion. B. Remove the saline lock from the client's arm. C. Increase the rate of the normal saline infusion. D. Change the IV solution to 0.45% saline solution.

Decrease the rate of the normal saline infusion. Rationale: For a client diagnosed with dehydration who is receiving IV fluids at 125mL/hour, a bounding pulse, tachycardia, and pedal edema are signs of fluid volume overload, so when contacting the healthcare provider, the nurse should anticipate a prescription for decreasing the rate of IV fluid administration.

A client with type 2 diabetes mellitus presents to the emergency room with signs of hyperosmolar hyperglycemic state (HHS). Which of the client's serum laboratory values requires intervention by the nurse? A. Low serum phosphate level. B. Elevated white blood cells. C. Decreased serum potassium. D. Positive urinary ketones.

Decreased serum potassium. Rationale: Hyperosmolar hyperglycemic state (HHS) is a complication of diabetes mellitus (predominantly type 2) in which high blood sugars cause severe dehydration, increases in osmolarity, and a high risk of complications such as coma and death. Low serum potassium can occur causing cardiac irritability which is life threatening.

The home health nurse makes a home visit to a male client with amyotrophic lateral sclerosis (ALS). The client is sitting upright while feeding himself and coughs frequently during the meal. What action should the nurse implement? A. Assist the client to lie down and turn to the side. B. Encourage the use of assistive feeding devices. C. Demonstrate use of a tucked-chin position while eating. D. Recommend the use of supplemental liquid feedings.

Demonstrate use of a tucked-chin position while eating. Rationale: Amyotrophic lateral sclerosis (ALS), an upper and lower motor degenerative disease, is characterized by progressive weakness that increases risk for aspiration while eating. For the alert client, sitting upright and using a tucked-chin position can help reduce risk for aspiration while maintaining the client's independence.

The nurse is assessing a middle-aged adult who is diagnosed with osteoarthritis. Which factor in this client's history is a contributor to the osteoarthritis? A. Long distance runner since high school. B. Lactose intolerant since childhood. C. Photosensitive to a drug currently taking. D. Recently treated for deep vein thrombosis.

Long distance runner since high school. Rationale: Osteoarthritis is a degenerative joint disease often caused by traumatic injury or repetitive stress to weight-bearing joints, such as high impact sports like running.

The nurse manager observes that a staff nurse does not know how to perform many client care skills, is unable to complete assignments in a timely manner, and often persuades another nurse who was a former classmate to complete assignments. How should the nurse manager respond to this situation? A. Assign the nurse's former classmate to another shift to prevent further dependency. B. Submit documentation of the staff nurse's performance to the state Board of Nursing. C. Terminate both the staff nurse and former classmate for their breach of professionalism. D. Meet with the staff nurse and implement a written contract for performance improvement.

Meet with the staff nurse and implement a written contract for performance improvement. Rationale: Clinical incompetence can be correctable once identified, if handled sensitively and with immediacy. A contract for performance improvement as a form of remediation may result in a highly competent and committed employee. By doing a contract, it helps guide the employee in identifying job performance expectations and also gives them a sense of empowerment as they become competent in their clinical skills.

An older client with a 3 day history of abdominal distention is admitted with a small bowel obstruction. The nurse inserts a nasogastric tube and attaches it to low intermittent suction. Which ongoing client assessment takes priority when providing care? A. Observe skin integrity. B. Measure abdominal girth. C. Monitor fluid balance. D. Auscultate bowel sounds.

Monitor fluid balance. Rationale: Bowel obstructions cause severe fluid and electrolyte imbalance that can be life-threatening and require fluid and electrolyte replacement. The nursing priority is monitoring fluid balance.

The nurse is planning care for a client with chronic kidney disease who is a resident at a long-term nursing facility. The client is anuric and has hemodialysis 3 times a week. Which intervention should the nurse include in the client's plan of care? A. Encourage intake of high potassium foods. B. Provide perineal skin barrier cream. C. Initiate toileting schedule. D. Monitor for signs of anemia.

Monitor for signs of anemia. Rationale: Erythropoiesis is impaired in clients with chronic renal failure due to impaired production of erythropoietin, a hormone produced by the kidney that simulates bone marrow production of red blood cells and platelets. Monitoring for signs of anemia should be included in the plan of care.

The nurse identifies an electrolyte imbalance, an elevated pulse rate, and a weight gain of 4.4 lbs (2 kg) in 24 hours for a client with chronic kidney disease. What intervention should the nurse include in the plan of care? A. Provide only distilled water. B. Monitor serum electrolytes daily. C. Perform range of motion exercises. D. Document abdominal girth.

Monitor serum electrolytes daily. Rationale: Chronic kidney disease (CKD) is a progressive, irreversible loss of kidney function, decreasing glomerular filtration rate (GFR), and the kidney's inability to excrete metabolic waste products and water, resulting in fluid overload, electrolyte imbalance, elevated pulse rate, and weight gain. The plan of care should include monitor serum electrolytes daily to evaluate serum potassium and calcium that can predispose to fatal cardiac dysrhythmias.

Following a cardiac catheterization, an adult client is sent to the cardiovascular unit. The nurse instructs the client to keep the affected leg immobile. Which intervention should the nurse plan to include in this client's plan of care? A. Monitor telemetry for dysrhythmias. B. Apply a sequential compression device. C. Ambulate once vital signs stable. D. Maintain NPO until bowel sounds return.

Monitor telemetry for dysrhythmias. Rationale: Care for a client post cardiac catheterization via femoral artery cannulation includes keeping the affected leg immobile to minimize the risk for bleeding. Monitoring for dysrhythmias should be included in the plan of care due to the risk of myocardial irritability from the catheter passage, manipulation, and dye injection that can cause possible fluttering sensation of heart.

An adult client with schizophrenia who has been noncompliant in taking oral antipsychotic medications refuses a prescribed IM medication. Which action should the nurse take? A. Ask for staff assistance with administering the injection. B. Administer an oral PRN medication for agitation. C. Explain that oral medications will no longer be required. D. Notify the healthcare provider of the client's refusal.

Notify the healthcare provider of the client's refusal. Rationale: The healthcare provider should be notified that the client refused the IM prescription. Forcing administration of the medication by requesting staff assistance is a violation of the client's rights. Trying to reason with the client and those with schizophrenia who are not in touch with reality is ineffective.

Two weeks post-burn, a male client with 40% deep partial-thickness injury continues to have open wounds and is now developing diarrhea. His blood pressure is 80/40 mmHg and his temperature is 96o F (35.6o C). Which action is most important for the nurse to take? A. Continue to monitor vital signs. B. Increase the room temperature. C. Assess the oxygen saturation. D. Notify the rapid response team.

Notify the rapid response team. Rationale: Hypotension, hypothermia, and diarrhea are findings associated with systemic gram-negative sepsis, so the rapid response team or emergency team should be notified.

Following a cardiac catheterization and placement of a stent in the right coronary artery, the nurse administers prasugrel, a platelet inhibitor, to the client. To monitor for adverse effects from the medication, which assessment is most important for the nurse to include in this clients plan of care? A. Observe color of urine. B. Check for pedal edema. C. Assess skin turgor. D. Measure body temperature.

Observe color of urine. Rationale: Prasugrel, a platelet inhibitor, can cause hemorrhage, so it is critical to monitor for signs and symptoms of bleeding, such as pink-tinged urine.

Which assessment should the home health nurse include during a routine home visit for a client who was discharged home with a suprapubic catheter? A. Assess perineal area. B. Measure abdominal girth. C. Palpate flank area. D. Observe insertion site.

Observe insertion site. Rationale: A suprapubic catheter is inserted into the bladder through an incision, placing the client at risk for infection. The nurse should observe the insertion site during home visits for any signs of inflammation

An older resident of an extended care facility has recurrent urinary tract infections. The nursing care plan includes the goal "Increase daily intake of fluids." What nursing intervention is most useful in assisting the resident to meet this goal? A. Record the client's intake and output every shift. B. Increase fluids provided with the client's meals. C. Maintain a full pitcher of water at the bedside. D. Offer a glass of fluid every hour while awake.

Offer a glass of fluid every hour while awake. Rationale: Offering fluids every hour while awake will remind the client to drink fluids and encourages fluid intake.

A client who was splashed with a chemical has both eyes covered with bandages. When assisting the client with eating, which intervention should the nurse instruct the unlicensed assistive personnel (UAP) to implement? A. Feed the client the entire meal. B. Provide with only finger foods. C. Orient the client to the location of the food on the plate. D. Ask family to visit during meal time to assist with feeding.

Orient the client to the location of the food on the plate. Rationale: For a client who has both eyes covered with bandages, food locations on the meal tray should be explained using references to the places on the face of a clock. Such an explanation, orients the visually challenged client to food items on the meal tray so that they can feed themselves, and self-feeding helps foster a feeling of independence.

In early septic shock states, what is the primary cause of hypotension? A. Peripheral vasoconstriction. B. Peripheral vasodilation. C. Cardiac failure. D. A vagal response.

Peripheral vasodilation. Rationale: Toxins released by bacteria in septic shock create massive peripheral vasodilation and increase microvascular permeability at the site of the bacterial invasion.

A client is admitted to the hospital after experiencing a stroke or cerebral vascular accident (CVA). The nurse should request a referral for speech therapy if the client exhibits which finding? A. Abnormal responses for cranial nerves I and II. B. Unilateral facial drooping. C. Persistent coughing while drinking. D. Inappropriate or exaggerated mood swings.

Persistent coughing while drinking. Rationale: After a stroke, clients may experience dysphagia and an impaired gag reflex that is evaluated by a speech pathology team. Coughing while drinking results from impaired swallowing and gag reflex, so a referral to a speech therapist is indicated to evaluate the coordination of oral movements associated with speech and deglutition (swallowing).

An older adult resident of an extended care facility receives a prescription for diphenhydramine 25 mg PO to treat generalized pruritus. Two hours after administration of the drug, the client continues to experience itching, is confused, and has an unsteady gait. Which action should the nurse implement first? A. Give a second dose of diphenhydramine. B. Lubricate the skin with an emollient. C. Place the client on fall precautions. D. Apply soft limb restraints to extremities.

Place the client on fall precautions. Rationale: The client's confusion and unsteady gait pose a safety risk, so fall precautions should be implemented. Diphenhydramine is probably the cause of the client's neurological changes, and a second dose is likely to intensify these symptoms.

The nurse is planning to assess a client's oxygen saturation to determine if additional oxygen is needed via nasal cannula. The client has bilateral below-the-knee amputations and radial pulses that are weak and thready. What action should the nurse take? A. Document that an accurate oxygen saturation reading cannot be obtained. B. Elevate the client's hands for five minutes prior to obtaining a reading from the finger. C. Increase the oxygen based on the client's breathing patterns and lung sounds. D. Place the oximeter clip on the earlobe to obtain the oxygen saturation reading.

Place the oximeter clip on the earlobe to obtain the oxygen saturation reading. Rationale: Pulse oximeter clips can be attached to the earlobe to obtain an accurate measurement of oxygen saturation.

A client who underwent an uncomplicated gastric bypass surgery is having difficulty with diet management. Which dietary instruction is most important for the nurse to explain to the client? A. Chew food slowly and thoroughly before attempting to swallow. B. Plan volume-controlled, evenly-spaced meals throughout the day. C. Sip fluids slowly with each meal and between meals. D. Eliminate or reduce intake of fatty and gas forming foods.

Plan volume-controlled, evenly-spaced meals throughout the day. Rationale: It is most important for the client who underwent an uncomplicated gastric bypass surgery to learn how to eat without damaging the surgical site and to keep the digestive system from dumping the food instead of digesting it. Eating volume-controlled and evenly-spaced meals throughout the day allows the client to feel full, avoid binging, and eliminate the possibility of eating too much at one time.

The nurse is preparing a 50 mL dose of 50% Dextrose IV for a client with insulin shock. How should the nurse administer the medication? A. Dilute the Dextrose in one liter of 0.9% Normal Saline solution. B. Mix the Dextrose in a 50 mL piggyback for a total volume of 100 mL. C. Push the undiluted Dextrose slowly through the currently infusing IV. D. Ask the pharmacist to add the Dextrose to a TPN solution.

Push the undiluted Dextrose slowly through the currently infusing IV. Rationale: To reverse life-threatening insulin shock, the nurse should administer the 50% Dextrose as a slow IV bolus through the currently infusing IV.

An adolescent client on a drug treatment unit becomes angry and pulls the refrigerator from the wall and then throws the microwave. After the client fails to respond to redirection, the healthcare provider prescribes restraints. Which assessment should the nurse include in the client's record while the client is in restraints? A. Responsiveness to painful stimuli. B. Pupils equal, round and reactive. C. Range-of-motion and circulation. D. Speech patterns and processes.

Range-of-motion and circulation. Rationale: While a client is in restraints, the nurse should assess and record findings related to range-of-motion and circulation, and these assessments should be performed at regular intervals.

Four hours after surgery, a client reports nausea and begins to vomit. The nurse notes that the client has a scopolamine transdermal patch applied behind the ear. Which action should the nurse take? A. Reposition the transdermal patch to the client's trunk. B. Remove the transdermal patch until the vomiting subsides. C. Notify the client's healthcare provider of the vomiting. D. Explain that this is a side effect of the medication in the patch.

Rationale: Transdermal scopolamine is used to prevent nausea and vomiting from anesthesia and surgery. The nurse should notify the healthcare provider if the medication is ineffective.

A client with a history of dementia has become increasingly confused at night and is picking at an abdominal surgical dressing and the tape securing the intravenous (IV) line. The abdominal dressing is no longer occlusive, and the IV insertion site is pink. Which intervention should the nurse implement? A. Replace the IV site with a smaller gauge. B. Redress the abdominal incision. C. Leave the lights on in the room at night. D. Apply soft bilateral wrist restraints.

Redress the abdominal incision. Rationale: An abdominal incision should be redressed using aseptic technique when it is no longer occlusive after a client has been picking at it. The IV site should be assessed to ensure that it has not been dislodged due to the client picking at the tape and a dressing reapplied, if needed.

The nurse finds a client at 33-weeks gestation in cardiac arrest. What adaptation to cardiopulmonary resuscitation (CPR) should the nurse implement? A. Apply oxygen by mask after opening the airway. B. Position a firm wedge to support pelvis and thorax at 30 degree tilt. C. Give continuous compression with a ventilation ratio at 20:3. D. Apply less compression force to reduce aspiration.

Position a firm wedge to support pelvis and thorax at 30 degree tilt. Rationale: For the pregnant woman in cardiac arrest, it is important to relieve aortocaval compression caused by the gravid uterus. Left lateral uterine displacement (LUD) should be maximized using a firm wedge to support the pelvis and thorax at a 30° tilt to optimize maternal hemodynamics during CPR. Maternal modifications should include ventilation with 100% oxygen. Pregnant adults should be resuscitated using a compression-ventilation ratio of 30:2, without interruption of continuous compressions. Effective chest compressions should be forceful rhythmic applications of pressure ("fast and hard") at 100 compressions/minute at the depth of 2 inches (5 cm) to generate myocardial and cerebral blood flow.

A male client admitted with chronic pulmonary obstruction disease (COPD) exacerbation is receiving assisted ventilation with continuous positive airway pressure (CPAP). His vital signs are: temperature 98.8 oF (37.1 oC), heart rate 118 beats/minute, respirations 46 breaths/minute, blood pressure 176/92 mmHg. While completing the pulmonary assessment, his oxygen saturation reading is 78% and he is difficult to arouse. Which action should the nurse implement? A. Increase the oxygen delivery by 10%. B. Administer PRN nebulizer treatment. C. Complete neurological assessment. D. Prepare for rapid sequence intubation.

Prepare for rapid sequence intubation. Rationale: The vital signs and pulmonary assessment indicate that the CPAP is not improving the client's respiratory condition and his respiratory distress is worsening. A more secure airway via rapid sequence intubation is indicated and should be implemented immediately. Increasing the oxygen may worsen the COPD.

During a return demonstration of teaching provided by the nurse, the daughter of a client administers her mother's eye drops by resting her dominant hand on her mother's forehead and dropping the medication into the conjunctival sac. Which action should the nurse take in response to this demonstration? A. Advise the daughter to keep her hand farther from her mother's eye. B. Remind the client to gently close her eyes after the eye drops are instilled. C. Instruct the mother to gently rub the affected eye to distribute the drops. D. Offer to demonstrate the eye drop procedure to the daughter one more time.

Remind the client to gently close her eyes after the eye drops are instilled. Rationale: To instill eye drops correctly, the dominant hand should be rested on the forehead, and the eye drops directed toward the conjunctival sac. The daughter placed her hand correctly and correctly directed the eye drops toward the conjunctival sac; she should then instruct her mother to gently close her eyes to help distribute the medication evenly.

The nurse instructs a client in the performance of pursed lip breathing. The client performs a return demonstration as seen in the video. What instruction should the nurse provide after observing the client's return demonstration? A. Remind the client to purse lips while breathing out rather than while breathing in. B. Demonstrate how to splint the abdomen and cough after blowing air out completely. C. Encourage the client to increase the length of time for each inhalation. D. Commend the client for successfully performing the pursed lip breathing.

Remind the client to purse lips while breathing out rather than while breathing in. Rationale: When performing pursed lip breathing, which is used to prevent alveolar collapse and improve ventilation, the client should inhale deeply and purse lips during a prolonged exhalation. The client in the video demonstrated pursing of the lips during inhalation, so the nurse should remind the client to purse lips during exhalation.

A client with hyperthyroidism has a serum calcium level of 13.5 mg/dL (3.375 mmol/L). Based on this information, which instruction should the nurse provide the unlicensed assistive personnel (UAP) who is assisting with the care of the client? A. Report a change in the client's level of consciousness. B. Remove the water pitcher and restrict oral fluid intake. C. Provide dairy-rich snacks, such as ice cream. D. Assist with ambulation to prevent bone loss.

Report a change in the client's level of consciousness. Rationale: Thyroid hormone stimulates calcium resorption (loss) from bones causing hypercalcemia (normal 9 to 10.5 mg/dL [ 2.25 to 2.625 mmol/L]) Hypercalcemia can result in altered mental status, which the unlicensed assistive personnel (UAP) should report immediately to the nurse if the client demonstrates an alteration in mental status. The UAP should also encourage the client to drink fluids as tolerated to promote excretion of excess calcium.

While caring for a client's postoperative dressing, the nurse observes purulent wound drainage. Previously, the wound was inflamed and tender but without drainage. Which is the most important action for the nurse to take? A. Determine if the drainage has an unpleasant odor. B. Cleanse the wound with a sterile saline solution. C. Monitor the client's white blood cell count (WBC). D. Request a culture and sensitivity of the wound.

Request a culture and sensitivity of the wound. Rationale: A client who has a postoperative dressing with purulent drainage may have an infected wound and the most important action for the nurse to take is request a wound culture and sensitivity.

The school nurse is called to the soccer field because a child has a nose bleed (epistaxis). In which position should the nurse place the child? A. Sitting up and leaning forward. B. Standing with the head leaning backward. C. Side-lying with the head slightly elevated. D. Supine with the legs raised.

Sitting up and leaning forward. Rationale: A child with a nose bleed should be instructed to sit up and lean forward while continuous pressure is applied to the nose with the thumb and forefinger for at least ten minutes.

One year after being discharged from the burn trauma unit, a client with a history of 40 percent full-thickness burns is admitted with bone pain and muscle weakness. Which intervention should the nurse include in this client's plan of care? A. Encourage progressive active range of motion. B. Teach need for dietary and supplemental vitamin D3. C. Explain the need for skin exposure to sunlight without sunscreen. D. Instruct the client in the use of muscle strengthening exercises.

Teach need for dietary and supplemental vitamin D3. Rationale: Burn injury results in the acute loss of bone as well as the development of progressive vitamin D deficiency because burn scar tissue and adjacent normal-appearing skin cannot convert normal quantities of the precursors for vitamin D3 that is synthesized from ultraviolet sun rays and needed for strong bones. Client's with a history of full thickness burns should increase their dietary resources of vitamin D and supplemental D3.

The wife of a client diagnosed with Parkinson's disease tells the nurse that her husband is having trouble swallowing and she is afraid he is going to choke. Which intervention should the nurse implement? A. Offer the wife assurance that difficulty with swallowing is usually temporary. B. Encourage the couple to consider insertion of a nasogastric tube for tube feedings. C. Teach the wife to thicken all liquids and serve primarily semi-solid foods. D. Instruct wife to give carbidopa-levodopa 30-minutes before each meal.

Teach the wife to thicken all liquids and serve primarily semi-solid foods. Rationale: Dysphagia is usually a chronic problem for the client with Parkinson's disease. A semi-solid diet with thick liquids is easier to swallow than solid foods.

Several clients on a busy antepartum unit are scheduled for procedures that require informed consent. Which situation should the nurse explore further before witnessing the client's signature on the consent form? A. A 15-year-old primigravida who has been self-supporting for the past 6 months. B. The obstetrician explained a procedure that a neurologist will perform. C. The client was medicated for pain with a narcotic analgesic IM 6 hours ago. D. The client is illiterate but verbalizes understanding and consent for the procedure.

The obstetrician explained a procedure that a neurologist will perform. Rationale: The individual who is ultimately responsible for the procedure should provide the information necessary for informed consent, so when an obstetrician explains the procedure scheduled to be performed by a neurologist this should be explored further by the nurse before witnessing the client's signature.The obstetrician explained a procedure that a neurologist will perform.

The nurse is collecting a heelstick blood specimen for a neonatal screen, which includes thyroxine (T4) and thyroid stimulating hormone (TSH) levels, prior to the discharge of a 2-day-old newborn. When the parents ask why these tests are being conducted, which explanation should the nurse provide? A. This technique is used for early detection of mental retardation. B. Dosages for thyroid replacement therapy will be determined by this test. C. This is a routine blood test required by law to screen for metabolic deficiencies. D. These lab values will provide data to anticipate delays in growth and development.

This is a routine blood test required by law to screen for metabolic deficiencies. Rationale: The nurse should explain that the blood test is routine and is mandated by state law. The newborn screening tests are mandatory in most of the 50 states and help to detect inherited disorders of metabolism that can lead to mental retardation if not treated, such as congenital hypothyroidism, phenylketonuria and galactosemia.

A client with a history of diabetes and coronary artery disease is admitted with shortness of breath, anxiety, and confusion. The client's blood pressure is 80/60 mmHg, heart rate 120 beats/minute with audible third and fourth heart sounds, and bibasilar crackles. The client's average urinary output is 5 mL/hour. Normal saline is infusing at 125 mL/hour with a secondary infusion of dopamine at 4 mcg/kg/minute per infusion pump. Which intervention should the nurse implement? A. Irrigate the indwelling urinary catheter. B. Prepare the client for external pacing. C. Obtain capillary blood glucose measurement. D. Titrate the dopamine infusion to raise the BP.

Titrate the dopamine infusion to raise the BP. Rationale: The client is experiencing cardiogenic shock and requires titration of the vasoactive infusion dopamine to increase the blood pressure. Cardiogenic shock is a condition is which the heart cannot pump enough blood to meet the body's needs.

A client who is admitted to the intensive care unit with a right chest tube attached to a THORA-SEAL™ chest drainage unit becomes increasingly anxious and complains of difficulty breathing. The nurse determines the client is tachypneic with absent breath sounds in the client's right lung fields. Which additional finding indicates that the client has developed a tension pneumothorax? A. Continuous bubbling in the water-seal chamber. B. Decreased bright red bloody drainage. C. Tachypnea with difficulty breathing. D. Tracheal deviation toward the left lung.

Tracheal deviation toward the left lung. Rationale: Tracheal deviation toward the unaffected left lung with absent breath sounds over the affected right lung are classic late signs of a tension pneumothorax. Tachypnea with difficulty breathing are early signs of respiratory distress, which can result from a variety of causes, including a pneumothorax.

The nurse plans to collect a 24-hour urine specimen for a creatinine clearance test. Which instruction should the nurse provide to the adult male client? A. Cleanse around the meatus, discard first portion of voiding, and collect the rest in a sterile bottle. B. Urinate at a specified time, discard this urine, and collect all subsequent urine during the next 24 hours. C. For the next 24 hours, notify nurse when the bladder is full, and the nurse will collect catheterized specimens. D. Urinate immediately into a urinal, and the lab will collect the specimen every 6 hours, for the next 24 hours.

Urinate at a specified time, discard this urine, and collect all subsequent urine during the next 24 hours. Rationale: Voiding, discarding the sample, and beginning the collection are the correct steps for collecting a 24-hour urine specimen. Discarding even one voided specimen during the collection invalidates the test.

Five days after surgical fixation of a fractured femur, a client suddenly reports chest pain and difficulty in breathing. The nurse suspects that the client may have had a pulmonary embolus. What action should the nurse take first? A. Notify the healthcare provider. B. Bring the emergency crash cart to the bedside. C. Provide supplemental oxygen. D. Prepare a continuous heparin infusion per protocol.

Provide supplemental oxygen. Rationale: The client's chest pain and dyspnea occurring several days after orthopedic surgery suggest a pulmonary embolus. Oxygen is administered immediately to relieve hypoxemia which could lead to a deterioration in the client's condition.

The nurse is initiating IV fluid replacement for a child who has dry, sticky mucous membranes, flushed skin, and a fever of 103.6 oF (39.7 oC). Laboratory findings indicate that the child has a serum sodium concentration of 156 mEq/L (156 mmol/L). Which physiologic mechanism contributes to this finding? A. The intravenous fluid replacement contains a hypertonic solution of sodium chloride. B. Urinary and gastrointestinal fluid loss reduce blood viscosity and stimulate thirst. C. Insensible loss of body fluids contributes to the hemoconcentration of serum solutes. D. Hypothalamic resetting of core body temperature causes vasodilation to reduce body heat.

Insensible loss of body fluids contributes to the hemoconcentration of serum solutes. Rationale: Fever causes insensible fluid loss, which contributes to fluid volume deficit and results in hemoconcentration of sodium (serum sodium greater than 150 mEq/L or mmol/L). Dehydration, which is manifested by dry, sticky mucous membranes, and flushed skin, is often managed by replacing lost fluids and electrolytes with IV fluids that contain varying concentrations of sodium chloride.

The nurse is assessing a client with pulmonary edema who is reporting two pillow orthopnea and paroxysmal nocturnal dyspnea. The nurse identifies crackles in all lung fields and use of accessory muscles. Which action should the nurse include in the client's plan of care? A. Administer the prescribed amiodarone immediately. B. Institute a daily fluid restriction while the client is in the hospital. C. Arrange a prescribed electrophysiology study (EPS) for the client. D. Assess the client's commitment to their daily exercise regimen.

Institute a daily fluid restriction while the client is in the hospital. Rationale: In pulmonary edema, pulmonary capillary congestion, usually due to left-sided heart failure, causes fluid accumulation in the alveoli that compromises gaseous exchange between lungs and blood. This is manifested by orthopnea, frothy pink sputum, crackles, paroxysmal nocturnal dyspnea, and pulsus alternans. The nurse should institute a daily fluid restriction while the client is in the hospital.

When should the nurse conduct an Allen's test? A. When pulmonary artery pressures are obtained. B. Prior to attempting a cardiac output calculation. C. To assess for presence of a deep vein thrombus in the leg. D. Just before arterial blood gasses are drawn peripherally.

Just before arterial blood gasses are drawn peripherally. Rationale: The Allen's test should be performed prior to puncturing the radial artery to obtain a blood gas specimen to determine patency of the ulnar artery in the selected extremity. To perform an Allen's test the client's hand is formed into a fist while the nurse compresses the ulnar artery. Compression continues while the fist is opened. If blood perfusion through the radial artery is adequate, the hand should flush and resume its normal pinkish coloration.

An older adult client is receiving a second unit of blood when the nurse enters the room and finds the client sitting up in bed. The client is dyspneic and seems confused. Lung auscultation reveals crackles in the bases of both lungs. Vital sign measurement reveals a rapid, bounding pulse and elevated blood pressure. After discontinuing the transfusion, which intervention should the nurse implement? A. Obtain a urine specimen. B. Monitor for hives and pruritus. C. Send the blood bag and blood tubing to the blood bank. D. Keep the IV access line intact for diuretic administration.

Keep the IV access line intact for diuretic administration. Rationale: The client is demonstrating signs of fluid overload from the transfusion which will be managed through the use of intravenous (IV) diuretics. In the hospital setting whenever a client's condition is considered to be labile requiring an IV access and presently has a functioning IV access in place, the standard of care is to saline lock the IV catheter if parental fluids are no longer necessary, so emergency IV access is available if needed.

A client with scleroderma (systemic sclerosis), an auto-immune collagen disease, experiences frequent severe pain caused by Raynaud's phenomenon. To help manage this problem, which instruction should the home health nurse provide the client and family? A. Apply cool compresses to swelling to alleviate pain. B. Elevate the feet whenever possible to prevent swelling. C. Keep the home environment warm to reduce episodes. D. Eat potassium-rich foods to decrease muscle cramping.

Keep the home environment warm to reduce episodes. Rationale: Severe pain caused by Raynaud's phenomenon is a common problem associated with scleroderma and occurs when exposure to cold or stress causes arteriolar constriction and resultant decreased blood flow to the digits. Maintaining a warm environment may help decrease episodes.


संबंधित स्टडी सेट्स

Chemistry Chapter 12: Stoichiometry

View Set

ITSE-1345 - Topic Quiz - Oracle PL/SQL Overview -1

View Set

Ch. 58 Care of Pts with Liver Problems

View Set

MACRO Chpt 20: International Finance

View Set

World Civ II Unit 2 Study Guide: Ming-Qing China and the Indian Mughal Dynasty

View Set